Diễn Đàn MathScopeDiễn Đàn MathScope
  Diễn Đàn MathScope
Ghi Danh Hỏi/Ðáp Thành Viên Social Groups Lịch Ðánh Dấu Ðã Ðọc

Go Back   Diễn Đàn MathScope > Sơ Cấp > Việt Nam và IMO > 2012

News & Announcements

Ngoài một số quy định đã được nêu trong phần Quy định của Ghi Danh , mọi người tranh thủ bỏ ra 5 phút để đọc thêm một số Quy định sau để khỏi bị treo nick ở MathScope nhé !

* Nội quy MathScope.Org

* Một số quy định chung !

* Quy định về việc viết bài trong diễn đàn MathScope

* Nếu bạn muốn gia nhập đội ngũ BQT thì vui lòng tham gia tại đây

* Những câu hỏi thường gặp

* Về việc viết bài trong Box Đại học và Sau đại học


Trả lời Gởi Ðề Tài Mới
 
Ðiều Chỉnh Xếp Bài
Old 15-01-2012, 09:43 PM   #16
kryptios
+Thành Viên+
 
kryptios's Avatar
 
Tham gia ngày: Oct 2010
Bài gởi: 64
Thanks: 20
Thanked 37 Times in 23 Posts
Thưa thầy, dùng pt pell như vậy nhưng quên không giả sử x>=y và tính toán sai công thức của dãy v(n) khi kết luận a,b thuộc dãy số thì bị trừ khoảng mấy điểm thầy
Và câu b bài 3 tính mấy điểm thầy
[RIGHT][I][B]Nguồn: MathScope.ORG[/B][/I][/RIGHT]
 
__________________
...kryptios is...kryptos..
kryptios is offline   Trả Lời Với Trích Dẫn
Old 15-01-2012, 10:38 PM   #17
Nts_pbc
+Thành Viên+
 
Tham gia ngày: Aug 2011
Đến từ: ..Yên Thành, Nghệ̣ An..BoxMath.vn..
Bài gởi: 28
Thanks: 11
Thanked 24 Times in 14 Posts
Thầy cho em hỏi bài 6 em làm đc đến k=4 thì được khoảng bao nhiêu điểm ạ?
[RIGHT][I][B]Nguồn: MathScope.ORG[/B][/I][/RIGHT]
 
Nts_pbc is offline   Trả Lời Với Trích Dẫn
Old 15-01-2012, 11:11 PM   #18
pvthuan
+Thành Viên+
 
Tham gia ngày: Jan 2008
Bài gởi: 81
Thanks: 18
Thanked 108 Times in 36 Posts
Trong link dưới đây là một số lời giải nữa. Hy vọng là mới.
[Only registered and activated users can see links. ]
[RIGHT][I][B]Nguồn: MathScope.ORG[/B][/I][/RIGHT]
 
__________________
hexagon.edu.vn
pvthuan is offline   Trả Lời Với Trích Dẫn
The Following User Says Thank You to pvthuan For This Useful Post:
hoangkhtn2010 (30-01-2012)
Old 16-01-2012, 03:40 PM   #19
tranvuxuannhat
+Thành Viên+
 
Tham gia ngày: Nov 2010
Đến từ: thpt trần quốc tuấn quảng ngãi
Bài gởi: 28
Thanks: 45
Thanked 10 Times in 7 Posts
Thưa thầy, nếu như em chỉ khẳng định hàm số tăng là đơn ánh (theo em đọc tài liệu cua nxb giáo dục thì phân ra là tăng và không giảm) thì em bi trừ bao nhiêu điểm ạ? em có xây dựng hàm ngược và giải ra được hàm f.
[RIGHT][I][B]Nguồn: MathScope.ORG[/B][/I][/RIGHT]
 
__________________
Cuộc sống không có kẻ thắng người bại, chỉ có hạnh phúc
tranvuxuannhat is offline   Trả Lời Với Trích Dẫn
Old 16-01-2012, 03:58 PM   #20
cr7_manhcuong
+Thành Viên+
 
Tham gia ngày: Aug 2010
Bài gởi: 3
Thanks: 10
Thanked 0 Times in 0 Posts
Cậu cứ yên tâm.nếu bài giải đúng thì mình nghĩ họ sẽ nương tay!
[RIGHT][I][B]Nguồn: MathScope.ORG[/B][/I][/RIGHT]
 
cr7_manhcuong is offline   Trả Lời Với Trích Dẫn
Old 16-01-2012, 06:03 PM   #21
caohiep
+Thành Viên+
 
Tham gia ngày: Nov 2011
Bài gởi: 4
Thanks: 3
Thanked 0 Times in 0 Posts
Thầy Dũng có thể đưa ra một dự đoán tương đối về điểm chuẩn của năm nay không ạ?
[RIGHT][I][B]Nguồn: MathScope.ORG[/B][/I][/RIGHT]
 
caohiep is offline   Trả Lời Với Trích Dẫn
Old 16-01-2012, 11:03 PM   #22
namdung
Administrator

 
Tham gia ngày: Feb 2009
Đến từ: Tp Hồ Chí Minh
Bài gởi: 1,343
Thanks: 209
Thanked 4,066 Times in 778 Posts
Gửi tin nhắn qua Yahoo chát tới namdung
Chào các bạn,

Có 3 việc tôi không thể trả lời chính xác:

1. Không làm a, không làm b sẽ bị trừ bao nhiêu điểm?
2. Chỉ làm a, chỉ làm b sẽ được bao nhiêu điểm?
3. Điểm chuẩn năm nay là bao nhiêu?

Hai việc đầu phải chờ đến khi thành lập Hội đồng giám khảo, các thầy thảo luận cụ thể thang điểm mới biết được.

Việc thứ hai càng khó, phải chấm xong mới biết được.

Chỉ có kinh nghiệm qua các kỳ thi và qua nghe ngóng tình hình thì đoán sơ bộ là năm nay ai làm 5 bài (3 + 2) thì có lẽ chắc ăn TST. Dưới đó thì còn phải chờ xem.

Riêng điểm đoạt giải KK chắc vẫn ở tầm 10-12 điểm thôi, cho dù năm nay có nhiều bài ngon ăn.
[RIGHT][I][B]Nguồn: MathScope.ORG[/B][/I][/RIGHT]
 
namdung is offline   Trả Lời Với Trích Dẫn
The Following 10 Users Say Thank You to namdung For This Useful Post:
AnhIsGod (05-02-2012), caohiep (17-01-2012), cr7_manhcuong (16-01-2012), dhth7b (17-01-2012), hizact (08-02-2012), hoangcongduc (19-01-2012), ngocson_dhsp (17-01-2012), Nts_pbc (17-01-2012), TKT (17-01-2012), YUGI_94_K51 (17-01-2012)
Old 19-01-2012, 05:25 PM   #23
namdung
Administrator

 
Tham gia ngày: Feb 2009
Đến từ: Tp Hồ Chí Minh
Bài gởi: 1,343
Thanks: 209
Thanked 4,066 Times in 778 Posts
Gửi tin nhắn qua Yahoo chát tới namdung
Tôi gửi lại bài bình luận có bổ sung

1. Bổ sung bình luận sau bài 1 về vấn đề công thức tổng quát và bài toán tổng quát.
2. Bổ sung bài tập tương tự sau bài 2.
3. Bổ sung lời giải bài 6 theo hướng sử dụng PT Pell.
4. Nói thêm về vấn đề: Có phải hàm tăng và toàn ánh thì liên tục, cũng như tác dụng của tính chất này đối với bài PTH.
[RIGHT][I][B]Nguồn: MathScope.ORG[/B][/I][/RIGHT]
 
File Kèm Theo
Kiểu File : doc VMO2012_LoigiaivaBinhluan.doc (623.0 KB, 285 lần tải)
namdung is offline   Trả Lời Với Trích Dẫn
The Following 9 Users Say Thank You to namdung For This Useful Post:
DaiToan (19-01-2012), hizact (19-01-2012), kien10a1 (19-01-2012), lovemath_ltv (23-01-2012), maxo (19-01-2012), minh_thương911 (19-01-2012), n.v.thanh (19-01-2012), thiendienduong (20-01-2012), thinhptnk (20-01-2012)
Old 19-01-2012, 07:00 PM   #24
minh_thương911
+Thành Viên+
 
minh_thương911's Avatar
 
Tham gia ngày: Dec 2011
Đến từ: Ninh Thuận
Bài gởi: 50
Thanks: 29
Thanked 49 Times in 28 Posts
Gửi tin nhắn qua Yahoo chát tới minh_thương911
Còn đây là file pdf cho đẹp ạ.
[RIGHT][I][B]Nguồn: MathScope.ORG[/B][/I][/RIGHT]
 
File Kèm Theo
Kiểu File : pdf VMO2012_LoigiaivaBinhluan.pdf (726.2 KB, 430 lần tải)
minh_thương911 is offline   Trả Lời Với Trích Dẫn
The Following 3 Users Say Thank You to minh_thương911 For This Useful Post:
linh1997 (12-01-2014), thiendienduong (20-01-2012), tranghieu95 (19-01-2012)
Old 19-01-2012, 07:46 PM   #25
tuandaisu
+Thành Viên+
 
Tham gia ngày: Jul 2009
Bài gởi: 9
Thanks: 6
Thanked 7 Times in 4 Posts
Xin hỏi các bạn trên MS rằng: Nếu bài 4 tôi dùng phép quy nạp thì có vấn đề gì không? Đây là một vấn đề mà tôi đang có sự tranh luận với một người về nó. Cách tôi chứng minh gồm hai bước:
1. Số keo lớn nhất đạt đựoc khi ở đầu mỗi hàng co1 bạn nam và 1 bạn nữ đứng cạnh nhau. (tương đối dễ)
2. Loại đi hai bạn đó thì số kẹo mà tổng số nhận được giảm đi n^2+n. (Cái này tôi chứng minh được nhưng chưa hài lòng với cách chứng minh đó.)

Và sau cùng tôi thu được dấu bằng xảy ra khi mà trong hai vị trí (2n-1;2n) có 1 bạn nam và 1 bạn nữ.

Mong các bạn xem thử liệu chứng minh như vậy có gì không chặt chẽ về mặt logic không? Tôi xin chân thành cám ơn.
[RIGHT][I][B]Nguồn: MathScope.ORG[/B][/I][/RIGHT]
 
tuandaisu is offline   Trả Lời Với Trích Dẫn
The Following User Says Thank You to tuandaisu For This Useful Post:
thiendienduong (22-01-2012)
Old 20-01-2012, 06:20 AM   #26
tuandaisu
+Thành Viên+
 
Tham gia ngày: Jul 2009
Bài gởi: 9
Thanks: 6
Thanked 7 Times in 4 Posts
Sau đây tôi xin trình bày cách giải (theo phương pháp quy nạp), chân thành mong các bạn góp ý nếu có sai sót gì.
Gọi S(n) là số kẹo dùng nhiều nhất cho bài toán có kích thước n.
Thấy n = 2 thì bài toán đúng.
Giả sử bài toán đúng với n. Ta sẽ chứng minh nó đúng với n + 1.
Bước 1: Giả sử có $k \geq 2 $ bạn nam đứng ở ngoài cùng bên phải. Ta tiến hành đổi chỗ bạn nam thứ 2 từ ngoài cùng bên phải với bạn nữ đứng cạnh k bạn nam trên. Sau khi đổi chỗ thì số kẹo của các bạn nam nữ khác k + 1 bạn đang xét không đổi. Số kẹo của k bạn nam này tăng lên là $(k-1)n $, số kẹo bạn nữ đang từ $(n+1-k)k $ thành n.
Vậy số kẹo sau khi chuyển đổi thay đổi: (k-1)n+n-(n+1-k)k=k(k-1)>0.
Như vậy sau khi thực hiện thao tác trên thì ta thu được số kẹo là tốt hơn thực sự.
Xét trường hợp sắp xếp sao cho hai bạn ngoài cùng bên phải khác giới nhau. Giả sử hai bạn đó sắp xếp: X....XNuNam
Bước 2: Loại bỏ hai bạn này đi thì số kẹo sẽ thay đổi như sau:
Bạn Nữ loại sẽ mất n kẹo.
Các bạn nam và bạn nữ còn lại mất số kẹo bằng số các bạn khác giới đứng bên trái mình. Và ta sẽ chứng minh tổng số kẹo này bằng $n^2 $.
Xét các tuyến nối bạn X bất kỳ thuộc hàng với bạn khác giới bên trái của mình. Như vậy tổng số các tuyến là $n^2 $ và ta thấy số tuyến có đầu phải từ X chính bằng số các bạn khác giới với X nằm ở bên trái X. Như vậy số kẹo mất đúng bằng $n^2+n $. 2n bạn còn lại thì sau khi "trả lại kẹo" thì số kẹo không vượt quá $S(n) $
Từ đó số kẹo cho 2n+2 bạn tối ưu là: $S(n+1)=S(n)+n^2+n=\frac {(n+1)[(n+1)^2-1]}{3} $

Qua cách chứng minh này ta có thể khẳng định là dấu bằng xảy ra khi và chỉ khi trong các khối dạng (2k-1;2k) có 1 bạn nam và 1 bạn nữ
[RIGHT][I][B]Nguồn: MathScope.ORG[/B][/I][/RIGHT]
 

thay đổi nội dung bởi: tuandaisu, 20-01-2012 lúc 11:05 PM
tuandaisu is offline   Trả Lời Với Trích Dẫn
The Following 2 Users Say Thank You to tuandaisu For This Useful Post:
thiendienduong (22-01-2012), thinhptnk (20-01-2012)
Old 22-01-2012, 03:19 PM   #27
namdung
Administrator

 
Tham gia ngày: Feb 2009
Đến từ: Tp Hồ Chí Minh
Bài gởi: 1,343
Thanks: 209
Thanked 4,066 Times in 778 Posts
Gửi tin nhắn qua Yahoo chát tới namdung
Tôi nghĩ về cơ bản bạn làm đúng rồi. Chỉ có trong bước 2 cần làm chi tiết hơn 1 chút.
[RIGHT][I][B]Nguồn: MathScope.ORG[/B][/I][/RIGHT]
 
namdung is offline   Trả Lời Với Trích Dẫn
The Following 2 Users Say Thank You to namdung For This Useful Post:
lovemath_ltv (23-01-2012), thiendienduong (22-01-2012)
Old 27-01-2012, 10:58 PM   #28
hoangcongduc
+Thành Viên+
 
Tham gia ngày: Dec 2009
Đến từ: THPT chuyên Lý Tự Trọng Cần Thơ
Bài gởi: 78
Thanks: 92
Thanked 64 Times in 41 Posts
Gửi tin nhắn qua Yahoo chát tới hoangcongduc
Thưa thầy, cho em hỏi, khi nào thì có kết quả thi VMO ạ ? Và kết quả được công bố ở đâu ạ ?
[RIGHT][I][B]Nguồn: MathScope.ORG[/B][/I][/RIGHT]
 
__________________
-----------------
-------------------------
TIÊN HỌC LỄ HẬU HỌC THÊM
hoangcongduc is offline   Trả Lời Với Trích Dẫn
Old 27-01-2012, 11:58 PM   #29
tuandaisu
+Thành Viên+
 
Tham gia ngày: Jul 2009
Bài gởi: 9
Thanks: 6
Thanked 7 Times in 4 Posts
Trích:
Nguyên văn bởi namdung View Post
Tôi nghĩ về cơ bản bạn làm đúng rồi. Chỉ có trong bước 2 cần làm chi tiết hơn 1 chút.
Em cám ơn anh đã góp ý cho lời giải của em. Thực ra đây chỉ là ý tưởng để giải quyết vấn đề và cũng để khẳng định rằng bài toán này hoàn toàn giải quyết gọn gàng bằng phương pháp quy nạp mà thôi. Đây cũng là một vấn đề mà em tranh cãi với một thầy từ năm ngoái. Một lần nữa chân thành cám ơn anh và chúc anh cùng gia đình một năm mới an khang - thịnh vượng. Chúc toàn thể MSers một năm mới vạn sự như ý.
[RIGHT][I][B]Nguồn: MathScope.ORG[/B][/I][/RIGHT]
 
tuandaisu is offline   Trả Lời Với Trích Dẫn
Old 28-01-2012, 08:08 AM   #30
ThangToan
+Thành Viên+
 
Tham gia ngày: Nov 2010
Đến từ: THPT chuyên Vĩnh Phúc
Bài gởi: 570
Thanks: 24
Thanked 537 Times in 263 Posts
Trích:
Nguyên văn bởi namdung View Post
Tôi gửi lại bài bình luận có bổ sung

1. Bổ sung bình luận sau bài 1 về vấn đề công thức tổng quát và bài toán tổng quát.
2. Bổ sung bài tập tương tự sau bài 2.
3. Bổ sung lời giải bài 6 theo hướng sử dụng PT Pell.
4. Nói thêm về vấn đề: Có phải hàm tăng và toàn ánh thì liên tục, cũng như tác dụng của tính chất này đối với bài PTH.
tôi xin trình bày nhận xét 4 mà nhiều người quan tâm như sau
với mọi dãy $\[\left( {{x_n}} \right)\]
$ sao cho $\[\mathop {\lim }\limits_{n \to \infty } {x_n} = a\]
$ ta sẽ chứng minh $\[\mathop {\lim }\limits_{n \to \infty } f\left( {{x_n}} \right) = f\left( a \right)\]
$ thật vậy
tồn tại số nguyên dương k đủ lớn sao cho
$\[a - \frac{1}{n} \le {x_n} \le a + \frac{1}{n},\forall n \ge k\]
$ [1]
từ bất đẳng thức [1] và f là hàm tăng suy ra bđt sau;
$\[f\left( {a - \frac{1}{n}} \right) \le f\left( {{x_n}} \right) \le f\left( {a + \frac{1}{n}} \right),\forall n \ge k\]
$ [2]
mặt khác ta có $\[\left( {a + \frac{1}{n}} \right)\]
$ là dãy giảm bị chặn dưới bởi $a $ nên dãy số $\[\left( {f\left( {a+ \frac{1}{n}} \right)} \right)\]
$ là dãy giảm bị chặn dưới bởi $\[f\left( a \right)\]
$ nên tồn tại $b $ sao cho $\[\lim f\left( {a - \frac{1}{n}} \right) = b\]
$. do $f $ là toàn ánh nên tồn tại $z $ sao cho
$\[b = f\left( z \right)\]
$
tương tự ta có $\[\left( {a - \frac{1}{n}} \right)\]
$ là dãy tăng bị chặn trên bởi $a $ nên dãy số $\[\left( {f\left( {a - \frac{1}{n}} \right)} \right)\]
$ là dãy tăng bị chặn trên bởi $\[f\left( a \right)\]
$ nên tồn tại $c $ sao cho $\[\lim f\left( {a - \frac{1}{n}} \right) = c\]
$. do $f $ là toàn ánh nên tồn tại $t $ sao cho
$\[c = f\left( t \right)\]
$
từ bất đẳng thức [2] ta được $\[f\left( z \right) \ge f\left( t \right) \Rightarrow z \ge t\]
$ và từ bất đẳng thức này, kết hợp với $\[\left( {a + \frac{1}{n}} \right)\]
$ là dãy giảm và $\[\left( {a - \frac{1}{n}} \right)\]
$ là dãy tăng ta suy ra được bđt sau;
$\[a + \frac{1}{n} \ge z \ge t \ge a - \frac{1}{n},\forall n \ge k\]
$ [3]
từ [3] chuyển qua giới hạn ta được $z=t=a $. do đó từ bđt [2] chuyển qua giới hạn ta được $\[f\left( z \right) \ge \mathop {\lim }\limits_{n \to \infty } f\left( {{x_n}} \right) = f\left( a \right) \ge f\left( t \right) = f\left( z \right)\]
$ hay $\[\mathop {\lim }\limits_{n \to \infty } f\left( {{x_n}} \right) = f\left( a \right)\]
$
từ đó suy ra hàm số $f $ liên tục.
[RIGHT][I][B]Nguồn: MathScope.ORG[/B][/I][/RIGHT]
 
ThangToan is offline   Trả Lời Với Trích Dẫn
The Following User Says Thank You to ThangToan For This Useful Post:
minh_thương911 (28-01-2012)
Trả lời Gởi Ðề Tài Mới

Bookmarks

Ðiều Chỉnh
Xếp Bài

Quuyền Hạn Của Bạn
You may not post new threads
You may not post replies
You may not post attachments
You may not edit your posts

BB code is Mở
Smilies đang Mở
[IMG] đang Mở
HTML đang Tắt

Chuyển đến


Múi giờ GMT. Hiện tại là 07:13 PM.


Powered by: vBulletin Copyright ©2000-2024, Jelsoft Enterprises Ltd.
Inactive Reminders By mathscope.org
[page compression: 101.34 k/116.80 k (13.23%)]